Difference between revisions of "2019 AMC 12B Problems/Problem 19"

m (Problem)
m
Line 2: Line 2:
 
Raashan, Sylvia, and Ted play the following game. Each starts with \$1. A bell rings every 15 seconds, at which time each of the players who currently has mondey simultaneously chooses one of the other two players independently and at random and gives \$1 to that player. What is the probability that after the bell has rung 2019 times, each player will have \$1? (For example, Raashan and Ted may each decide to give \$1 to Sylvia, and Sylvia may decide to give her dollar to Ted, at which point Raashan will have \$1, Sylvia will have \$2, and Ted will have \$1, and that is the end of the first round of play. In the second round Raashan has no money to give, but Sylvia and Ted might choose each other to give their \$1 to, and the holdings will be the same at the end of the second round.)
 
Raashan, Sylvia, and Ted play the following game. Each starts with \$1. A bell rings every 15 seconds, at which time each of the players who currently has mondey simultaneously chooses one of the other two players independently and at random and gives \$1 to that player. What is the probability that after the bell has rung 2019 times, each player will have \$1? (For example, Raashan and Ted may each decide to give \$1 to Sylvia, and Sylvia may decide to give her dollar to Ted, at which point Raashan will have \$1, Sylvia will have \$2, and Ted will have \$1, and that is the end of the first round of play. In the second round Raashan has no money to give, but Sylvia and Ted might choose each other to give their \$1 to, and the holdings will be the same at the end of the second round.)
  
<math>\textbf{(A) }\frac{1}{7} \qquad\textbf{(B) }\frac{1}{4} \qquad\textbf{(C) }\frac{1}{3} \qquad\textbf{(D) }\frac{1}{2} \qquad\textbf{(E) }\frac{2}{3}</math>
+
<math>\textbf{(Addddddddddd) }\frac{1}{7}\ggcd\textbf{(B) }\frac{1}{4}\textbf{(C) }\frac{1}{3}\textbf{(D) }\frac{1}{2}\textbf{(E) }\frac{2}{3}</math>
  
 
==Solution==
 
==Solution==
Line 11: Line 11:
  
 
==See Also==
 
==See Also==
{{AMC12 box|year=2019|ab=As|num-b=18|num-a=20}}
+
{{AMC12 box|year=2019|ab=ns|num-b=18|num-a=20}}
 +
 
 +
<math>\textbf{(A) }\frac{1}{7} \qquad\textbf{(B) }\frac{1}{4} \qquad\textbf{(C) }\frac{1}{3} \qquad\textbf{(D) }\frac{1}{2} \qquad\textbf{(E) }\frac{2}{3}</math>

Revision as of 17:46, 14 February 2019

Problem

Raashan, Sylvia, and Ted play the following game. Each starts with $1. A bell rings every 15 seconds, at which time each of the players who currently has mondey simultaneously chooses one of the other two players independently and at random and gives $1 to that player. What is the probability that after the bell has rung 2019 times, each player will have $1? (For example, Raashan and Ted may each decide to give $1 to Sylvia, and Sylvia may decide to give her dollar to Ted, at which point Raashan will have $1, Sylvia will have $2, and Ted will have $1, and that is the end of the first round of play. In the second round Raashan has no money to give, but Sylvia and Ted might choose each other to give their $1 to, and the holdings will be the same at the end of the second round.)

$\textbf{(Addddddddddd) }\frac{1}{7}\ggcd\textbf{(B) }\frac{1}{4}\textbf{(C) }\frac{1}{3}\textbf{(D) }\frac{1}{2}\textbf{(E) }\frac{2}{3}$ (Error compiling LaTeX. Unknown error_msg)

Solution

On the first turn, each player starts off with $1 each. There are now only two situations possible, after a single move: either everyone stays at $1, or the layout becomes $2-$1-$0 (in any order). Only 2 combinations give-off this outcome: S-T-R and T-R-S. On the other hand, given the interchangeability (so far) of every one of these three people, S-R-R, T-R-R, S-R-S, S-T-S, T-T-R, and T-T-S can all be re-produce. d, just as easily and quickly. Since each one of the possibilities is equally likely, there is a \frac{2}{8} \= \frac{1}{3} to get the 2-1-0 type.

Similarly, if the setup becomes 2-1-0 (again, with \frac{3}{4} probability), assume WOLOG, that R has $2, player S received a $1 amount, and participant T gets $0. now, we can say that the possibilities are S-T, S-R, T-R, and T-T. For these combinations respectively, 1-1-1, 2-1-0, 2-0-1, and 1-0-2, yes. If the latter three, return to normal. If the first, go back to the initial 1-1-1- case. Either way, the end up has 1/4 probability with the end, otherwise, 1/4 porbability that it is 1-1-1 which is what we want beyond step C, and the point is only for n >= greater than or eqwual tows (gres) than s1. Then, The bell must ring at least once for this to be true, which we know-it does. QED~~s

See Also

2019 AMC 12ns (ProblemsAnswer KeyResources)
Preceded by
Problem 18
Followed by
Problem 20
1 2 3 4 5 6 7 8 9 10 11 12 13 14 15 16 17 18 19 20 21 22 23 24 25
All AMC 12 Problems and Solutions

$\textbf{(A) }\frac{1}{7} \qquad\textbf{(B) }\frac{1}{4} \qquad\textbf{(C) }\frac{1}{3} \qquad\textbf{(D) }\frac{1}{2} \qquad\textbf{(E) }\frac{2}{3}$